Answer to Question #159038 in Field Theory for zain ul abdeen

Question #159038

. An electron is accelerated through 2 400 V from rest and then enters a uniform 1.70-T magnetic field. What are (a) the maximum and (b) the minimum values of the magnetic force this particle can experience? 


1
Expert's answer
2021-01-27T16:49:21-0500

a)


"qU=0.5mv^2\\\\v=\\sqrt{\\frac{2qU}{m}}\\\\F=qvB\n\\\\F=qB\\sqrt{\\frac{2qU}{m}}\\\\F=(1.6\\cdot10^{-19})(1.7)\\sqrt{\\frac{2(2400)(1.6\\cdot10^{-19})}{(9.1\\cdot10^{-31})}}\\\\=7.9\\cdot10^{-12}\\ N"

b) If the magnetic field is parallel to the motion of the electron:


"F=0\\ N"


Need a fast expert's response?

Submit order

and get a quick answer at the best price

for any assignment or question with DETAILED EXPLANATIONS!

Comments

No comments. Be the first!

Leave a comment

LATEST TUTORIALS
New on Blog
APPROVED BY CLIENTS